• Matéria: Matemática
  • Autor: joseferreira855
  • Perguntado 7 anos atrás

Como resolver o sistema: {x^y=y^x ex^3=y^2 para x e y pertencente a R. Por favor. ajudem-me.


DanJR: Excelente questão!!

Respostas

respondido por: DanJR
1

Olá José!

Resposta:

\boxed{\mathsf{S = \left \{ \left ( \frac{9}{4}, \frac{27}{8} \right ), \left ( 1, 1 \right ) \right \}}}

Explicação passo-a-passo:

Resolvamos o sistema abaixo:

\begin{cases} \mathtt{x^y = y^x \qquad \qquad \qquad (i)} \\ \mathtt{x^3 = y^2 \qquad \qquad \qquad (ii)} \end{cases}


Inicialmente, dividamos a resolução em duas condições:

\\ \mathtt{\bullet \qquad x \neq y} \\ \mathtt{\bullet \qquad x = y}


Isto posto,

\mathtt{\bullet \qquad \texttt{Condi\c c\~ao} \ I: \ x \neq y}


Da equação (i),

\\ \displaystyle \mathsf{x^y = y^x} \\\\ \mathsf{\log x^y = \log y^x} \\\\ \boxed{\mathsf{y \cdot \log x = x \cdot \log y}} \qquad \qquad \mathtt{(iii)}


Da equação (ii),

\\ \displaystyle \mathsf{x^3 = y^2} \\\\ \mathsf{\log x^3 = \log y^2} \\\\ \mathsf{3 \cdot \log x = 2 \cdot \log y} \\\\ \boxed{\mathsf{\log x = \frac{2}{3} \cdot \log y}} \qquad \qquad \mathtt{(iv)}


Obs.: de acordo com a definição de logaritmos, temos:

\mathtt{x, y > 0}


Por conseguinte, e, levando em consideração a observação acima, substituímos a equação obtida em (iv) na equação (iii). Segue,

\\ \displaystyle \mathsf{y \cdot \log x = x \cdot \log y} \\\\ \mathsf{y \cdot \left ( \frac{2}{3} \cdot \cancel \log y \right ) = x \cdot \cancel \log y} \\\\ \boxed{\mathsf{x = \frac{2y}{3}}} \qquad \qquad \qquad \mathtt{(v)}


Ora, substituindo-o numa das equação iniciais do sistema, ou seja, nas equações (i) ou (ii), teremos que:

\\ \displaystyle \mathsf{x^3 = y^2} \\\\ \mathsf{\left ( \frac{2y}{3} \right )^3 = y^2} \\\\ \mathsf{\frac{8y^3}{27} = y^2} \\\\ \mathsf{8y^3 = 27y^2} \\\\ \mathsf{8y^3 - 27y^2 = 0} \\\\ \mathsf{y^2(8y - 27) = 0} \\\\ \boxed{\mathsf{S_y = \left \{ 0, \frac{27}{8} \right \}}}


Entretanto, ZERO não poderá ser considerada uma raiz pelo motivo visto na OBSERVAÇÃO acima!


Com efeito, determinamos a incógnita x, veja:

De (v),

\\ \displaystyle \mathsf{x = \frac{2y}{3}} \\\\\\ \mathsf{x = \frac{2}{3} \cdot \frac{27}{8}} \\\\\\ \mathsf{x = \frac{2}{3} \cdot \frac{3 \cdot 9}{2 \cdot 4}} \\\\\\ \boxed{\boxed{\mathsf{S_x = \left \{ \frac{9}{4} \right \}}}}


Feito isto, tiramos que quando x e y são distintos temos como solução o seguinte par ordenado:

\boxed{\boxed{\boxed{\boxed{\mathsf{S_1 = \left \{ \left ( \frac{9}{4}, \frac{27}{8} \right ) \right \}}}}}}


Ademais,

\mathtt{\bullet \qquad \texttt{Condi\c c\~ao} \ II: \ x = y}


Da equação (i), tiramos apenas que \mathtt{x, y > 0}; todavia, da equação (ii), concluímos que:

\\ \displaystyle \mathsf{x^3 = y^2} \\\\ \mathsf{x^3 = x^2} \\\\ \mathsf{x^3 - x^2 = 0} \\\\ \mathsf{x^2(x - 1) = 0} \\\\ \boxed{\mathsf{S_x = \left \{ 0, 1 \right \}}}


Uma vez que, nem x nem y podem ser nulo, concluímos que:

\boxed{\boxed{\boxed{\boxed{\mathsf{S_2 = \left \{ \left ( 1, 1 \right ) \right \}}}}}}



joseferreira855: Muito obriado!!! É que a resposta do livro consta também o par (0,0). Talvez o autor não considerou a OBS que você fez (x,y)>0. Mais valeu mesmo!
DanJR: José, obrigado por trazer essa informação!
DanJR: Estou a pensar se a 'observação' é pertinente...
DanJR: Bom! Parece-me que sim!!
joseferreira855: Valeu!!!
Perguntas similares